Integration using the Feynman Trick












4














I have a feeling this post won't met the community guidelines (will delete if so).



I'm looking for definite integrals that are solvable using the method of differentiation under the integral sign (also called the Feynman Trick) in order to practice using this technique.



Does anyone know of any good ones to tackle?










share|cite|improve this question
























  • @Q the Platypus - thanks for the edit.
    – DavidG
    Nov 7 '18 at 1:56
















4














I have a feeling this post won't met the community guidelines (will delete if so).



I'm looking for definite integrals that are solvable using the method of differentiation under the integral sign (also called the Feynman Trick) in order to practice using this technique.



Does anyone know of any good ones to tackle?










share|cite|improve this question
























  • @Q the Platypus - thanks for the edit.
    – DavidG
    Nov 7 '18 at 1:56














4












4








4


4





I have a feeling this post won't met the community guidelines (will delete if so).



I'm looking for definite integrals that are solvable using the method of differentiation under the integral sign (also called the Feynman Trick) in order to practice using this technique.



Does anyone know of any good ones to tackle?










share|cite|improve this question















I have a feeling this post won't met the community guidelines (will delete if so).



I'm looking for definite integrals that are solvable using the method of differentiation under the integral sign (also called the Feynman Trick) in order to practice using this technique.



Does anyone know of any good ones to tackle?







integration






share|cite|improve this question















share|cite|improve this question













share|cite|improve this question




share|cite|improve this question








edited Nov 7 '18 at 1:54









Q the Platypus

2,754933




2,754933










asked Nov 7 '18 at 1:46









DavidG

1,836619




1,836619












  • @Q the Platypus - thanks for the edit.
    – DavidG
    Nov 7 '18 at 1:56


















  • @Q the Platypus - thanks for the edit.
    – DavidG
    Nov 7 '18 at 1:56
















@Q the Platypus - thanks for the edit.
– DavidG
Nov 7 '18 at 1:56




@Q the Platypus - thanks for the edit.
– DavidG
Nov 7 '18 at 1:56










5 Answers
5






active

oldest

votes


















5














Here are some that I have encountered:
$$I_1=int_0^frac{pi}{2} ln(sec^2x +tan^4x)dx$$
$$I_2=int_0^infty frac{lnleft({1+x+x^2}right)}{1+x^2}dx$$
$$I_3=int_0^frac{pi}{2}ln(2+tan^2x)dx$$
$$I_4=int_0^infty frac{x-sin x}{x^3(x^2+4)} dx$$
$$I_5=int_0^frac{pi}{2}arcsinleft(frac{sin x}{sqrt 2}right)dx$$
$$I_6=int_0^frac{pi}{2} lnleft(frac{2+sin x}{2-sin x}right)dx$$
$$I_7=int_0^frac{pi}{2} frac{arctan(sin x)}{sin x}dx $$
$$I_8=int_0^1 frac{ln(1+x^3)}{1+x^2}dx $$
$$I_9=int_0^{infty} frac{x^{4/5}-x^{2/3}}{ln(x)(1+x^2)}dx$$
$$I_{10}=int_0^1 frac{ln(1+x)}{x(1+x^2)}dx$$
In case you struggle where to put that parameter, feel free to ask.






share|cite|improve this answer



















  • 1




    I am working my way through the list (thanks again). I solved $I_4$ yesterday math.stackexchange.com/questions/3004469/…
    – DavidG
    Nov 20 '18 at 4:16






  • 1




    That is nice to hear! I will try to add more when I have time.
    – Zacky
    Nov 20 '18 at 9:59






  • 1




    Have been coming back and forth with these. Got $I_7$ out yesterday. Was very close to $I_1$ too! math.stackexchange.com/questions/3024869/…
    – DavidG
    Dec 3 '18 at 23:27






  • 1




    That is nice to hear, as for $,I_1=int_0^frac{pi}{2} ln(sec^2x +tan^4x)dx,$ I tried to keep it in the original way, but after the substitution $tan x =t$ might be clearer how to use Feynman's trick.
    – Zacky
    Dec 4 '18 at 0:40






  • 1




    I finally got $I_1$ out! math.stackexchange.com/questions/3026362/…
    – DavidG
    Dec 4 '18 at 23:46



















3














A few good ones are:
$$int_0^infty e^{-frac{x^2}{y^2}-y^2}dx$$
$$int_0^infty frac{1-cos(xy)}xdx$$
$$int_0^infty frac{dx}{(x^2+p)^{n+1}}$$
$$int_{0}^{infty}e^{-x^2}dx$$
$$int_0^infty cos(x^2)dx$$
$$int_0^infty sin(x^2)dx$$
$$int_0^infty frac{sin^2x}{x^2(x^2+1)}dx$$
$$int_0^{pi/2} xcot x dx$$
That should keep you busy for a while ;)






share|cite|improve this answer





















  • @clatharus - Legend!! thanks very much!
    – DavidG
    Nov 7 '18 at 3:30










  • @clatharus - I've solved two of these already :-) math.stackexchange.com/questions/2950772/… math.stackexchange.com/questions/2966938/…
    – DavidG
    Nov 7 '18 at 3:32



















2














Maybe you can look at:



https://math.stackexchange.com/a/2989801/186817



Feynman's trick is used to compute:



begin{align}int_0^{frac{pi}{12}}ln(tan x),dxend{align}






share|cite|improve this answer





















  • Is the upper limit meant to be $frac{pi}{2}$
    – DavidG
    Nov 16 '18 at 1:00










  • No, it's $dfrac{pi}{12}$. with upper bound to be $dfrac{pi}{2}$: begin{align}int_0^{frac{pi}{2}}ln(tan x),dx=0end{align} (perform the change of variable $y=dfrac{pi}{2}-x$ )
    – FDP
    Nov 16 '18 at 15:28



















1














you can try the most famous one which is:
$$int_0^inftyfrac{sin(x)}{x}dx$$
good luck!






share|cite|improve this answer





















  • Yes, have solved that one! Thanks though :-)
    – DavidG
    Nov 7 '18 at 1:52






  • 1




    How about this: math.stackexchange.com/questions/2918366/…
    – Henry Lee
    Nov 7 '18 at 1:53



















1














Another example is in evaluating
$$displaystyle int_0^infty dfrac{cos xdx}{1+x^2}$$



by first considering
$$Ileft(aright)=int_{0}^{infty}frac{sinleft(axright)}{xleft(1+x^{2}right)}dx,,a>0$$ we have $$I'left(aright)=int_{0}^{infty}frac{cosleft(axright)}{1+x^{2}}dx$$
From which it can be shown that
$$Ileft(aright)=frac{pi}{2}left(1-e^{-a}right)$$
hence
$$lim_{arightarrow1}I'left(aright)=frac{pi }{2e}.$$






share|cite|improve this answer





















    Your Answer





    StackExchange.ifUsing("editor", function () {
    return StackExchange.using("mathjaxEditing", function () {
    StackExchange.MarkdownEditor.creationCallbacks.add(function (editor, postfix) {
    StackExchange.mathjaxEditing.prepareWmdForMathJax(editor, postfix, [["$", "$"], ["\\(","\\)"]]);
    });
    });
    }, "mathjax-editing");

    StackExchange.ready(function() {
    var channelOptions = {
    tags: "".split(" "),
    id: "69"
    };
    initTagRenderer("".split(" "), "".split(" "), channelOptions);

    StackExchange.using("externalEditor", function() {
    // Have to fire editor after snippets, if snippets enabled
    if (StackExchange.settings.snippets.snippetsEnabled) {
    StackExchange.using("snippets", function() {
    createEditor();
    });
    }
    else {
    createEditor();
    }
    });

    function createEditor() {
    StackExchange.prepareEditor({
    heartbeatType: 'answer',
    autoActivateHeartbeat: false,
    convertImagesToLinks: true,
    noModals: true,
    showLowRepImageUploadWarning: true,
    reputationToPostImages: 10,
    bindNavPrevention: true,
    postfix: "",
    imageUploader: {
    brandingHtml: "Powered by u003ca class="icon-imgur-white" href="https://imgur.com/"u003eu003c/au003e",
    contentPolicyHtml: "User contributions licensed under u003ca href="https://creativecommons.org/licenses/by-sa/3.0/"u003ecc by-sa 3.0 with attribution requiredu003c/au003e u003ca href="https://stackoverflow.com/legal/content-policy"u003e(content policy)u003c/au003e",
    allowUrls: true
    },
    noCode: true, onDemand: true,
    discardSelector: ".discard-answer"
    ,immediatelyShowMarkdownHelp:true
    });


    }
    });














    draft saved

    draft discarded


















    StackExchange.ready(
    function () {
    StackExchange.openid.initPostLogin('.new-post-login', 'https%3a%2f%2fmath.stackexchange.com%2fquestions%2f2987994%2fintegration-using-the-feynman-trick%23new-answer', 'question_page');
    }
    );

    Post as a guest















    Required, but never shown

























    5 Answers
    5






    active

    oldest

    votes








    5 Answers
    5






    active

    oldest

    votes









    active

    oldest

    votes






    active

    oldest

    votes









    5














    Here are some that I have encountered:
    $$I_1=int_0^frac{pi}{2} ln(sec^2x +tan^4x)dx$$
    $$I_2=int_0^infty frac{lnleft({1+x+x^2}right)}{1+x^2}dx$$
    $$I_3=int_0^frac{pi}{2}ln(2+tan^2x)dx$$
    $$I_4=int_0^infty frac{x-sin x}{x^3(x^2+4)} dx$$
    $$I_5=int_0^frac{pi}{2}arcsinleft(frac{sin x}{sqrt 2}right)dx$$
    $$I_6=int_0^frac{pi}{2} lnleft(frac{2+sin x}{2-sin x}right)dx$$
    $$I_7=int_0^frac{pi}{2} frac{arctan(sin x)}{sin x}dx $$
    $$I_8=int_0^1 frac{ln(1+x^3)}{1+x^2}dx $$
    $$I_9=int_0^{infty} frac{x^{4/5}-x^{2/3}}{ln(x)(1+x^2)}dx$$
    $$I_{10}=int_0^1 frac{ln(1+x)}{x(1+x^2)}dx$$
    In case you struggle where to put that parameter, feel free to ask.






    share|cite|improve this answer



















    • 1




      I am working my way through the list (thanks again). I solved $I_4$ yesterday math.stackexchange.com/questions/3004469/…
      – DavidG
      Nov 20 '18 at 4:16






    • 1




      That is nice to hear! I will try to add more when I have time.
      – Zacky
      Nov 20 '18 at 9:59






    • 1




      Have been coming back and forth with these. Got $I_7$ out yesterday. Was very close to $I_1$ too! math.stackexchange.com/questions/3024869/…
      – DavidG
      Dec 3 '18 at 23:27






    • 1




      That is nice to hear, as for $,I_1=int_0^frac{pi}{2} ln(sec^2x +tan^4x)dx,$ I tried to keep it in the original way, but after the substitution $tan x =t$ might be clearer how to use Feynman's trick.
      – Zacky
      Dec 4 '18 at 0:40






    • 1




      I finally got $I_1$ out! math.stackexchange.com/questions/3026362/…
      – DavidG
      Dec 4 '18 at 23:46
















    5














    Here are some that I have encountered:
    $$I_1=int_0^frac{pi}{2} ln(sec^2x +tan^4x)dx$$
    $$I_2=int_0^infty frac{lnleft({1+x+x^2}right)}{1+x^2}dx$$
    $$I_3=int_0^frac{pi}{2}ln(2+tan^2x)dx$$
    $$I_4=int_0^infty frac{x-sin x}{x^3(x^2+4)} dx$$
    $$I_5=int_0^frac{pi}{2}arcsinleft(frac{sin x}{sqrt 2}right)dx$$
    $$I_6=int_0^frac{pi}{2} lnleft(frac{2+sin x}{2-sin x}right)dx$$
    $$I_7=int_0^frac{pi}{2} frac{arctan(sin x)}{sin x}dx $$
    $$I_8=int_0^1 frac{ln(1+x^3)}{1+x^2}dx $$
    $$I_9=int_0^{infty} frac{x^{4/5}-x^{2/3}}{ln(x)(1+x^2)}dx$$
    $$I_{10}=int_0^1 frac{ln(1+x)}{x(1+x^2)}dx$$
    In case you struggle where to put that parameter, feel free to ask.






    share|cite|improve this answer



















    • 1




      I am working my way through the list (thanks again). I solved $I_4$ yesterday math.stackexchange.com/questions/3004469/…
      – DavidG
      Nov 20 '18 at 4:16






    • 1




      That is nice to hear! I will try to add more when I have time.
      – Zacky
      Nov 20 '18 at 9:59






    • 1




      Have been coming back and forth with these. Got $I_7$ out yesterday. Was very close to $I_1$ too! math.stackexchange.com/questions/3024869/…
      – DavidG
      Dec 3 '18 at 23:27






    • 1




      That is nice to hear, as for $,I_1=int_0^frac{pi}{2} ln(sec^2x +tan^4x)dx,$ I tried to keep it in the original way, but after the substitution $tan x =t$ might be clearer how to use Feynman's trick.
      – Zacky
      Dec 4 '18 at 0:40






    • 1




      I finally got $I_1$ out! math.stackexchange.com/questions/3026362/…
      – DavidG
      Dec 4 '18 at 23:46














    5












    5








    5






    Here are some that I have encountered:
    $$I_1=int_0^frac{pi}{2} ln(sec^2x +tan^4x)dx$$
    $$I_2=int_0^infty frac{lnleft({1+x+x^2}right)}{1+x^2}dx$$
    $$I_3=int_0^frac{pi}{2}ln(2+tan^2x)dx$$
    $$I_4=int_0^infty frac{x-sin x}{x^3(x^2+4)} dx$$
    $$I_5=int_0^frac{pi}{2}arcsinleft(frac{sin x}{sqrt 2}right)dx$$
    $$I_6=int_0^frac{pi}{2} lnleft(frac{2+sin x}{2-sin x}right)dx$$
    $$I_7=int_0^frac{pi}{2} frac{arctan(sin x)}{sin x}dx $$
    $$I_8=int_0^1 frac{ln(1+x^3)}{1+x^2}dx $$
    $$I_9=int_0^{infty} frac{x^{4/5}-x^{2/3}}{ln(x)(1+x^2)}dx$$
    $$I_{10}=int_0^1 frac{ln(1+x)}{x(1+x^2)}dx$$
    In case you struggle where to put that parameter, feel free to ask.






    share|cite|improve this answer














    Here are some that I have encountered:
    $$I_1=int_0^frac{pi}{2} ln(sec^2x +tan^4x)dx$$
    $$I_2=int_0^infty frac{lnleft({1+x+x^2}right)}{1+x^2}dx$$
    $$I_3=int_0^frac{pi}{2}ln(2+tan^2x)dx$$
    $$I_4=int_0^infty frac{x-sin x}{x^3(x^2+4)} dx$$
    $$I_5=int_0^frac{pi}{2}arcsinleft(frac{sin x}{sqrt 2}right)dx$$
    $$I_6=int_0^frac{pi}{2} lnleft(frac{2+sin x}{2-sin x}right)dx$$
    $$I_7=int_0^frac{pi}{2} frac{arctan(sin x)}{sin x}dx $$
    $$I_8=int_0^1 frac{ln(1+x^3)}{1+x^2}dx $$
    $$I_9=int_0^{infty} frac{x^{4/5}-x^{2/3}}{ln(x)(1+x^2)}dx$$
    $$I_{10}=int_0^1 frac{ln(1+x)}{x(1+x^2)}dx$$
    In case you struggle where to put that parameter, feel free to ask.







    share|cite|improve this answer














    share|cite|improve this answer



    share|cite|improve this answer








    edited Dec 23 '18 at 19:56

























    answered Nov 15 '18 at 21:02









    Zacky

    5,0761752




    5,0761752








    • 1




      I am working my way through the list (thanks again). I solved $I_4$ yesterday math.stackexchange.com/questions/3004469/…
      – DavidG
      Nov 20 '18 at 4:16






    • 1




      That is nice to hear! I will try to add more when I have time.
      – Zacky
      Nov 20 '18 at 9:59






    • 1




      Have been coming back and forth with these. Got $I_7$ out yesterday. Was very close to $I_1$ too! math.stackexchange.com/questions/3024869/…
      – DavidG
      Dec 3 '18 at 23:27






    • 1




      That is nice to hear, as for $,I_1=int_0^frac{pi}{2} ln(sec^2x +tan^4x)dx,$ I tried to keep it in the original way, but after the substitution $tan x =t$ might be clearer how to use Feynman's trick.
      – Zacky
      Dec 4 '18 at 0:40






    • 1




      I finally got $I_1$ out! math.stackexchange.com/questions/3026362/…
      – DavidG
      Dec 4 '18 at 23:46














    • 1




      I am working my way through the list (thanks again). I solved $I_4$ yesterday math.stackexchange.com/questions/3004469/…
      – DavidG
      Nov 20 '18 at 4:16






    • 1




      That is nice to hear! I will try to add more when I have time.
      – Zacky
      Nov 20 '18 at 9:59






    • 1




      Have been coming back and forth with these. Got $I_7$ out yesterday. Was very close to $I_1$ too! math.stackexchange.com/questions/3024869/…
      – DavidG
      Dec 3 '18 at 23:27






    • 1




      That is nice to hear, as for $,I_1=int_0^frac{pi}{2} ln(sec^2x +tan^4x)dx,$ I tried to keep it in the original way, but after the substitution $tan x =t$ might be clearer how to use Feynman's trick.
      – Zacky
      Dec 4 '18 at 0:40






    • 1




      I finally got $I_1$ out! math.stackexchange.com/questions/3026362/…
      – DavidG
      Dec 4 '18 at 23:46








    1




    1




    I am working my way through the list (thanks again). I solved $I_4$ yesterday math.stackexchange.com/questions/3004469/…
    – DavidG
    Nov 20 '18 at 4:16




    I am working my way through the list (thanks again). I solved $I_4$ yesterday math.stackexchange.com/questions/3004469/…
    – DavidG
    Nov 20 '18 at 4:16




    1




    1




    That is nice to hear! I will try to add more when I have time.
    – Zacky
    Nov 20 '18 at 9:59




    That is nice to hear! I will try to add more when I have time.
    – Zacky
    Nov 20 '18 at 9:59




    1




    1




    Have been coming back and forth with these. Got $I_7$ out yesterday. Was very close to $I_1$ too! math.stackexchange.com/questions/3024869/…
    – DavidG
    Dec 3 '18 at 23:27




    Have been coming back and forth with these. Got $I_7$ out yesterday. Was very close to $I_1$ too! math.stackexchange.com/questions/3024869/…
    – DavidG
    Dec 3 '18 at 23:27




    1




    1




    That is nice to hear, as for $,I_1=int_0^frac{pi}{2} ln(sec^2x +tan^4x)dx,$ I tried to keep it in the original way, but after the substitution $tan x =t$ might be clearer how to use Feynman's trick.
    – Zacky
    Dec 4 '18 at 0:40




    That is nice to hear, as for $,I_1=int_0^frac{pi}{2} ln(sec^2x +tan^4x)dx,$ I tried to keep it in the original way, but after the substitution $tan x =t$ might be clearer how to use Feynman's trick.
    – Zacky
    Dec 4 '18 at 0:40




    1




    1




    I finally got $I_1$ out! math.stackexchange.com/questions/3026362/…
    – DavidG
    Dec 4 '18 at 23:46




    I finally got $I_1$ out! math.stackexchange.com/questions/3026362/…
    – DavidG
    Dec 4 '18 at 23:46











    3














    A few good ones are:
    $$int_0^infty e^{-frac{x^2}{y^2}-y^2}dx$$
    $$int_0^infty frac{1-cos(xy)}xdx$$
    $$int_0^infty frac{dx}{(x^2+p)^{n+1}}$$
    $$int_{0}^{infty}e^{-x^2}dx$$
    $$int_0^infty cos(x^2)dx$$
    $$int_0^infty sin(x^2)dx$$
    $$int_0^infty frac{sin^2x}{x^2(x^2+1)}dx$$
    $$int_0^{pi/2} xcot x dx$$
    That should keep you busy for a while ;)






    share|cite|improve this answer





















    • @clatharus - Legend!! thanks very much!
      – DavidG
      Nov 7 '18 at 3:30










    • @clatharus - I've solved two of these already :-) math.stackexchange.com/questions/2950772/… math.stackexchange.com/questions/2966938/…
      – DavidG
      Nov 7 '18 at 3:32
















    3














    A few good ones are:
    $$int_0^infty e^{-frac{x^2}{y^2}-y^2}dx$$
    $$int_0^infty frac{1-cos(xy)}xdx$$
    $$int_0^infty frac{dx}{(x^2+p)^{n+1}}$$
    $$int_{0}^{infty}e^{-x^2}dx$$
    $$int_0^infty cos(x^2)dx$$
    $$int_0^infty sin(x^2)dx$$
    $$int_0^infty frac{sin^2x}{x^2(x^2+1)}dx$$
    $$int_0^{pi/2} xcot x dx$$
    That should keep you busy for a while ;)






    share|cite|improve this answer





















    • @clatharus - Legend!! thanks very much!
      – DavidG
      Nov 7 '18 at 3:30










    • @clatharus - I've solved two of these already :-) math.stackexchange.com/questions/2950772/… math.stackexchange.com/questions/2966938/…
      – DavidG
      Nov 7 '18 at 3:32














    3












    3








    3






    A few good ones are:
    $$int_0^infty e^{-frac{x^2}{y^2}-y^2}dx$$
    $$int_0^infty frac{1-cos(xy)}xdx$$
    $$int_0^infty frac{dx}{(x^2+p)^{n+1}}$$
    $$int_{0}^{infty}e^{-x^2}dx$$
    $$int_0^infty cos(x^2)dx$$
    $$int_0^infty sin(x^2)dx$$
    $$int_0^infty frac{sin^2x}{x^2(x^2+1)}dx$$
    $$int_0^{pi/2} xcot x dx$$
    That should keep you busy for a while ;)






    share|cite|improve this answer












    A few good ones are:
    $$int_0^infty e^{-frac{x^2}{y^2}-y^2}dx$$
    $$int_0^infty frac{1-cos(xy)}xdx$$
    $$int_0^infty frac{dx}{(x^2+p)^{n+1}}$$
    $$int_{0}^{infty}e^{-x^2}dx$$
    $$int_0^infty cos(x^2)dx$$
    $$int_0^infty sin(x^2)dx$$
    $$int_0^infty frac{sin^2x}{x^2(x^2+1)}dx$$
    $$int_0^{pi/2} xcot x dx$$
    That should keep you busy for a while ;)







    share|cite|improve this answer












    share|cite|improve this answer



    share|cite|improve this answer










    answered Nov 7 '18 at 2:29









    clathratus

    3,243331




    3,243331












    • @clatharus - Legend!! thanks very much!
      – DavidG
      Nov 7 '18 at 3:30










    • @clatharus - I've solved two of these already :-) math.stackexchange.com/questions/2950772/… math.stackexchange.com/questions/2966938/…
      – DavidG
      Nov 7 '18 at 3:32


















    • @clatharus - Legend!! thanks very much!
      – DavidG
      Nov 7 '18 at 3:30










    • @clatharus - I've solved two of these already :-) math.stackexchange.com/questions/2950772/… math.stackexchange.com/questions/2966938/…
      – DavidG
      Nov 7 '18 at 3:32
















    @clatharus - Legend!! thanks very much!
    – DavidG
    Nov 7 '18 at 3:30




    @clatharus - Legend!! thanks very much!
    – DavidG
    Nov 7 '18 at 3:30












    @clatharus - I've solved two of these already :-) math.stackexchange.com/questions/2950772/… math.stackexchange.com/questions/2966938/…
    – DavidG
    Nov 7 '18 at 3:32




    @clatharus - I've solved two of these already :-) math.stackexchange.com/questions/2950772/… math.stackexchange.com/questions/2966938/…
    – DavidG
    Nov 7 '18 at 3:32











    2














    Maybe you can look at:



    https://math.stackexchange.com/a/2989801/186817



    Feynman's trick is used to compute:



    begin{align}int_0^{frac{pi}{12}}ln(tan x),dxend{align}






    share|cite|improve this answer





















    • Is the upper limit meant to be $frac{pi}{2}$
      – DavidG
      Nov 16 '18 at 1:00










    • No, it's $dfrac{pi}{12}$. with upper bound to be $dfrac{pi}{2}$: begin{align}int_0^{frac{pi}{2}}ln(tan x),dx=0end{align} (perform the change of variable $y=dfrac{pi}{2}-x$ )
      – FDP
      Nov 16 '18 at 15:28
















    2














    Maybe you can look at:



    https://math.stackexchange.com/a/2989801/186817



    Feynman's trick is used to compute:



    begin{align}int_0^{frac{pi}{12}}ln(tan x),dxend{align}






    share|cite|improve this answer





















    • Is the upper limit meant to be $frac{pi}{2}$
      – DavidG
      Nov 16 '18 at 1:00










    • No, it's $dfrac{pi}{12}$. with upper bound to be $dfrac{pi}{2}$: begin{align}int_0^{frac{pi}{2}}ln(tan x),dx=0end{align} (perform the change of variable $y=dfrac{pi}{2}-x$ )
      – FDP
      Nov 16 '18 at 15:28














    2












    2








    2






    Maybe you can look at:



    https://math.stackexchange.com/a/2989801/186817



    Feynman's trick is used to compute:



    begin{align}int_0^{frac{pi}{12}}ln(tan x),dxend{align}






    share|cite|improve this answer












    Maybe you can look at:



    https://math.stackexchange.com/a/2989801/186817



    Feynman's trick is used to compute:



    begin{align}int_0^{frac{pi}{12}}ln(tan x),dxend{align}







    share|cite|improve this answer












    share|cite|improve this answer



    share|cite|improve this answer










    answered Nov 8 '18 at 10:50









    FDP

    4,92911324




    4,92911324












    • Is the upper limit meant to be $frac{pi}{2}$
      – DavidG
      Nov 16 '18 at 1:00










    • No, it's $dfrac{pi}{12}$. with upper bound to be $dfrac{pi}{2}$: begin{align}int_0^{frac{pi}{2}}ln(tan x),dx=0end{align} (perform the change of variable $y=dfrac{pi}{2}-x$ )
      – FDP
      Nov 16 '18 at 15:28


















    • Is the upper limit meant to be $frac{pi}{2}$
      – DavidG
      Nov 16 '18 at 1:00










    • No, it's $dfrac{pi}{12}$. with upper bound to be $dfrac{pi}{2}$: begin{align}int_0^{frac{pi}{2}}ln(tan x),dx=0end{align} (perform the change of variable $y=dfrac{pi}{2}-x$ )
      – FDP
      Nov 16 '18 at 15:28
















    Is the upper limit meant to be $frac{pi}{2}$
    – DavidG
    Nov 16 '18 at 1:00




    Is the upper limit meant to be $frac{pi}{2}$
    – DavidG
    Nov 16 '18 at 1:00












    No, it's $dfrac{pi}{12}$. with upper bound to be $dfrac{pi}{2}$: begin{align}int_0^{frac{pi}{2}}ln(tan x),dx=0end{align} (perform the change of variable $y=dfrac{pi}{2}-x$ )
    – FDP
    Nov 16 '18 at 15:28




    No, it's $dfrac{pi}{12}$. with upper bound to be $dfrac{pi}{2}$: begin{align}int_0^{frac{pi}{2}}ln(tan x),dx=0end{align} (perform the change of variable $y=dfrac{pi}{2}-x$ )
    – FDP
    Nov 16 '18 at 15:28











    1














    you can try the most famous one which is:
    $$int_0^inftyfrac{sin(x)}{x}dx$$
    good luck!






    share|cite|improve this answer





















    • Yes, have solved that one! Thanks though :-)
      – DavidG
      Nov 7 '18 at 1:52






    • 1




      How about this: math.stackexchange.com/questions/2918366/…
      – Henry Lee
      Nov 7 '18 at 1:53
















    1














    you can try the most famous one which is:
    $$int_0^inftyfrac{sin(x)}{x}dx$$
    good luck!






    share|cite|improve this answer





















    • Yes, have solved that one! Thanks though :-)
      – DavidG
      Nov 7 '18 at 1:52






    • 1




      How about this: math.stackexchange.com/questions/2918366/…
      – Henry Lee
      Nov 7 '18 at 1:53














    1












    1








    1






    you can try the most famous one which is:
    $$int_0^inftyfrac{sin(x)}{x}dx$$
    good luck!






    share|cite|improve this answer












    you can try the most famous one which is:
    $$int_0^inftyfrac{sin(x)}{x}dx$$
    good luck!







    share|cite|improve this answer












    share|cite|improve this answer



    share|cite|improve this answer










    answered Nov 7 '18 at 1:51









    Henry Lee

    1,773218




    1,773218












    • Yes, have solved that one! Thanks though :-)
      – DavidG
      Nov 7 '18 at 1:52






    • 1




      How about this: math.stackexchange.com/questions/2918366/…
      – Henry Lee
      Nov 7 '18 at 1:53


















    • Yes, have solved that one! Thanks though :-)
      – DavidG
      Nov 7 '18 at 1:52






    • 1




      How about this: math.stackexchange.com/questions/2918366/…
      – Henry Lee
      Nov 7 '18 at 1:53
















    Yes, have solved that one! Thanks though :-)
    – DavidG
    Nov 7 '18 at 1:52




    Yes, have solved that one! Thanks though :-)
    – DavidG
    Nov 7 '18 at 1:52




    1




    1




    How about this: math.stackexchange.com/questions/2918366/…
    – Henry Lee
    Nov 7 '18 at 1:53




    How about this: math.stackexchange.com/questions/2918366/…
    – Henry Lee
    Nov 7 '18 at 1:53











    1














    Another example is in evaluating
    $$displaystyle int_0^infty dfrac{cos xdx}{1+x^2}$$



    by first considering
    $$Ileft(aright)=int_{0}^{infty}frac{sinleft(axright)}{xleft(1+x^{2}right)}dx,,a>0$$ we have $$I'left(aright)=int_{0}^{infty}frac{cosleft(axright)}{1+x^{2}}dx$$
    From which it can be shown that
    $$Ileft(aright)=frac{pi}{2}left(1-e^{-a}right)$$
    hence
    $$lim_{arightarrow1}I'left(aright)=frac{pi }{2e}.$$






    share|cite|improve this answer


























      1














      Another example is in evaluating
      $$displaystyle int_0^infty dfrac{cos xdx}{1+x^2}$$



      by first considering
      $$Ileft(aright)=int_{0}^{infty}frac{sinleft(axright)}{xleft(1+x^{2}right)}dx,,a>0$$ we have $$I'left(aright)=int_{0}^{infty}frac{cosleft(axright)}{1+x^{2}}dx$$
      From which it can be shown that
      $$Ileft(aright)=frac{pi}{2}left(1-e^{-a}right)$$
      hence
      $$lim_{arightarrow1}I'left(aright)=frac{pi }{2e}.$$






      share|cite|improve this answer
























        1












        1








        1






        Another example is in evaluating
        $$displaystyle int_0^infty dfrac{cos xdx}{1+x^2}$$



        by first considering
        $$Ileft(aright)=int_{0}^{infty}frac{sinleft(axright)}{xleft(1+x^{2}right)}dx,,a>0$$ we have $$I'left(aright)=int_{0}^{infty}frac{cosleft(axright)}{1+x^{2}}dx$$
        From which it can be shown that
        $$Ileft(aright)=frac{pi}{2}left(1-e^{-a}right)$$
        hence
        $$lim_{arightarrow1}I'left(aright)=frac{pi }{2e}.$$






        share|cite|improve this answer












        Another example is in evaluating
        $$displaystyle int_0^infty dfrac{cos xdx}{1+x^2}$$



        by first considering
        $$Ileft(aright)=int_{0}^{infty}frac{sinleft(axright)}{xleft(1+x^{2}right)}dx,,a>0$$ we have $$I'left(aright)=int_{0}^{infty}frac{cosleft(axright)}{1+x^{2}}dx$$
        From which it can be shown that
        $$Ileft(aright)=frac{pi}{2}left(1-e^{-a}right)$$
        hence
        $$lim_{arightarrow1}I'left(aright)=frac{pi }{2e}.$$







        share|cite|improve this answer












        share|cite|improve this answer



        share|cite|improve this answer










        answered Nov 8 '18 at 11:02









        Kevin

        5,420822




        5,420822






























            draft saved

            draft discarded




















































            Thanks for contributing an answer to Mathematics Stack Exchange!


            • Please be sure to answer the question. Provide details and share your research!

            But avoid



            • Asking for help, clarification, or responding to other answers.

            • Making statements based on opinion; back them up with references or personal experience.


            Use MathJax to format equations. MathJax reference.


            To learn more, see our tips on writing great answers.





            Some of your past answers have not been well-received, and you're in danger of being blocked from answering.


            Please pay close attention to the following guidance:


            • Please be sure to answer the question. Provide details and share your research!

            But avoid



            • Asking for help, clarification, or responding to other answers.

            • Making statements based on opinion; back them up with references or personal experience.


            To learn more, see our tips on writing great answers.




            draft saved


            draft discarded














            StackExchange.ready(
            function () {
            StackExchange.openid.initPostLogin('.new-post-login', 'https%3a%2f%2fmath.stackexchange.com%2fquestions%2f2987994%2fintegration-using-the-feynman-trick%23new-answer', 'question_page');
            }
            );

            Post as a guest















            Required, but never shown





















































            Required, but never shown














            Required, but never shown












            Required, but never shown







            Required, but never shown

































            Required, but never shown














            Required, but never shown












            Required, but never shown







            Required, but never shown







            Popular posts from this blog

            Can a sorcerer learn a 5th-level spell early by creating spell slots using the Font of Magic feature?

            Does disintegrating a polymorphed enemy still kill it after the 2018 errata?

            A Topological Invariant for $pi_3(U(n))$